LSAT and Law School Admissions Forum

Get expert LSAT preparation and law school admissions advice from PowerScore Test Preparation.

Homework or Lesson help relating to our Accelerated or Live Online Accelerated Courses.
 artcast58
  • Posts: 6
  • Joined: May 28, 2015
|
#18786
On Question 1 logic game on pg 234- the rules to do not state that 'o' also come before 'L' as it is shown on page 342
Thus one can have:
o,p,l
p,o,l
p,l,o
this was my thinking and caused a wrong answer Quest 12
 artcast58
  • Posts: 6
  • Joined: May 28, 2015
|
#18787
excuse me it was #13. I understand why I got the answer wrong but I do not understand how they came up with that logic.
 Jon Denning
PowerScore Staff
  • PowerScore Staff
  • Posts: 904
  • Joined: Apr 11, 2011
|
#18837
Hey Art,

Thanks for the question (and clarification). O doesn't have to come before L, in fact O could go 7th in this game. O can also go last for question 13. What you know is that with K in 4 and P > L to the right (east) of K (from the rules), then when question 13 also puts O to the right (east) of K, spaces 5, 6, and 7 are filled by the group P, L, and O with P to the left (west) of L.

That of course leaves N, R, and M to the left (west) of K in spaces 1, 2, and 3, respectively: M is always next to K so must be 3, R cannot be first so must be 2, leaving N to go 1.

The order looks like this:

N R M K (P > L, O), with the first four spots fixed and some movement possible in the last three.

The only answer choice pairing that must be adjacent then is N and R, answer choice (D).

I hope that helps!

Get the most out of your LSAT Prep Plus subscription.

Analyze and track your performance with our Testing and Analytics Package.